Med Surg unit 4 chpt 39, 40, 41

¡Supera tus tareas y exámenes ahora con Quizwiz!

2.When preparing a patient for a capsule endoscopy study, the nurse should A) Ensure the patient understands the required bowel preparation. B) Have the patient return to the procedure room for removal of the capsule. C) Teach the patient to maintain a clear liquid diet throughout the procedure. D) Explain to the patient that conscious sedation will be used during placement of the capsule.

0.0/1.0 Correct Answer(s): A A capsule endoscopy study involves the patient performing a bowel prep to cleanse the bowel before swallowing the capsule. The patient will be on a clear liquid diet for 1 to 2 days before the procedure and will remain NPO for 4 to 6 hours after swallowing the capsule. The capsule is disposable and will pass naturally with the bowel movement. Points Earned:

10.The nurse is assisting a diabetic patient to learn dietary planning as part of initial management of diabetes. The nurse would encourage the patient to limit intake of which of the following foods to help reduce the percent of fat in the diet? A) Cheese B) Broccoli C) Chicken D) Oranges

0.0/1.0 Correct Answer(s): A Cheese is a product derived from animal sources and is higher in fat and calories than vegetables, fruit, and poultry. Points Earned:

1.When assessing a patient's abdomen, it would be most appropriate to A) Auscultate the abdomen before palpation. B) Palpate the abdomen before auscultation. C) Percuss the abdomen before auscultation. D) Perform deep palpation before light palpation.

0.0/1.0 Correct Answer(s): A During examination of the abdomen, auscultation is done before percussion and palpation because these latter procedures may alter the bowel sounds. Points Earned:

3.A patient has sought care because of a loss of 25 lb over the past 6 months, during which the patient claims to have made no significant dietary changes. The nurse should assess the patient for potential A) Thyroid disorders. B) Diabetes insipidus. C) Pituitary dysfunction. D) Parathyroid dysfunction.

0.0/1.0 Correct Answer(s): A Hyperthyroidism is associated with weight loss. Alterations in pituitary function, such as diabetes insipidus, and parathyroid dysfunction are not commonly associated with this phenomenon. Points Earned:

5.The nurse is assigned to the care of a 64-year-old patient diagnosed with type 2 diabetes. In formulating a teaching plan that encourages the patient to actively participate in management of the diabetes, which of the following should be the nurse's initial intervention? A) Assess patient's perception of what it means to have diabetes. B) Ask the patient to write down current knowledge about diabetes. C) Set goals for the patient to actively participate in managing his diabetes. D) Assume responsibility for all of the patient's care to decrease stress level.

0.0/1.0 Correct Answer(s): A In order for teaching to be effective, the first step is to assess the patient. Teaching can be individualized once the nurse is aware of what a diagnosis of diabetes means to the patient. Points Earned:

5.Which of the following methods should be used to confirm the placement of a patient's newly-inserted nasogastric (NG) feeding tube? A) X-ray B) Aspiration C) Air auscultation D) Measurement of external length

0.0/1.0 Correct Answer(s): A It is imperative to ensure that an NG tube is situated in the GI tract rather than the patient's lungs. When an NG tube has been recently inserted, it is important to confirm this placement with an x-ray that will identify the tube's radiopaque tip. Points Earned:

2.The nurse is caring for a patient who is 5'6" tall and weighs 186 lb. The nurse has discussed reasonable weight loss goals and a low-calorie diet with the patient. Which of these statements made by the patient indicates a need for further teaching? A) "I will limit intake to 500 calories a day." B) "I will try to eat very slowly during mealtimes." C) "I'll try to pick foods from all of the basic food groups." D) "It's important for me to begin a regular exercise program."

0.0/1.0 Correct Answer(s): A Limiting intake to 500 calories per day is not indicated for this patient and the severe calorie energy restriction would place this patient at risk for multiple nutrient deficiencies. Low-calorie diets are defined as having between 800 and 1200 calories per day. Points Earned:

11.Lab results are back for a 54-year-old patient with a 15-year history of diabetes. Which of the following lab results follows the expected pattern accompanying macrovascular disease as a complication of diabetes? A) Increased triglyceride levels B) Decreased low-density lipoproteins C) Increased high-density lipoproteins D) Decreased very-low-density lipoproteins

0.0/1.0 Correct Answer(s): A Macrovascular complications of diabetes include changes to large- and medium-sized blood vessels. They include cerebrovascular, cardiovascular, and peripheral vascular disease. Increased triglyceride levels are associated with these macrovascular changes. For this reason, the patient should limit the amount of fat in the diet. Points Earned:

1.During the initial postoperative period following bariatric surgery, the nurse recognizes the importance of monitoring obese patients for respiratory insufficiency based on knowledge that A) The body stores anesthetics in adipose tissue. B) Intubation may be difficult because of extra chin skin folds. C) Postoperative pain may cause a decreased respiratory rate. D) The patient's head must remain flat for a minimum of 2 hours postprocedure.

0.0/1.0 Correct Answer(s): A The body stores anesthetics in adipose tissue, placing patients with excess adipose tissue at risk for resedation. As adipose cells release anesthetics back into the bloodstream, the patient may become sedated after surgery, increasing the risk of hypoventilation and resultant respiratory insufficiency. Points Earned:

7.The nurse is evaluating a 45-year-old patient diagnosed with type 2 diabetes mellitus. Which of the following symptoms reported by the patient is considered one of the classic clinical manifestations of diabetes? A) Excessive thirst B) Gradual weight gain C) Overwhelming fatigue D) Recurrent blurred vision

0.0/1.0 Correct Answer(s): A The classic symptoms of diabetes are polydipsia (excessive thirst), polyuria, (excessive urine output), and polyphagia (increased hunger).

4.The surgeon was unable to save a patient's parathyroid gland during a radical thyroidectomy. The nurse should consequently pay particular attention to which of the following components of the patient's laboratory values? A) Calcium levels B) Potassium levels C) Blood glucose levels D) Sodium and chloride levels

0.0/1.0 Correct Answer(s): A The parathyroid gland plays a key role in maintaining calcium levels. Potassium, sodium, glucose, and chloride are not directly influenced by the loss of the parathyroid gland. Points Earned:

3.The nurse has been teaching a patient with diabetes mellitus how to perform self-monitoring of blood glucose (SMBG). During evaluation of the patient's technique, the nurse identifies a need for additional teaching when the patient does which of the following? A) Chooses a puncture site in the center of the finger pad B) Washes hands with soap and water to cleanse the site to be used C) Warms the finger before puncturing the finger to obtain a drop of blood D) Tells the nurse that the result of 120 mg/dl indicates good control of diabetes

0.0/1.0 Correct Answer(s): A The patient should select a site on the sides of the fingertips, not on the center of the finger pad. This area contains many nerve endings and would be unnecessarily painful.

3.The nurse is caring for a 45-year-old woman with a herniated lumbar disk and obesity. The patient realizes that weight loss is necessary to lessen back strain. The patient is 5'6" tall and weighs 186 lb (84.5 kg) with a body mass index (BMI) of 28 kg/m2. The nurse explains to the patient that this measurement places her in which of the following weight categories? A) Normal weight B) Overweight C) Obese D) Morbidly obese

0.0/1.0 Correct Answer(s): B A normal BMI is 18.5 to 24.9 kg/m2, whereas a BMI of 25 to 29.9 kg/m2 is considered overweight. Points Earned:

2.Which of the following assessment parameters is of highest priority when caring for a patient undergoing a water deprivation test? A) Serum glucose B) Patient weight C) Arterial blood gases D) Patient temperature

0.0/1.0 Correct Answer(s): B A patient is at risk for severe dehydration during a water deprivation test. The test should be discontinued and the patient rehydrated if the patient's weight drops more than 2 kg at any time. The other assessment parameters do not assess fluid balance. Points Earned:

7.A patient who has suffered severe burns in a motor vehicle accident will soon be started on parenteral nutrition (PN). Which of the following principles should guide the nurse's administration of the patient's nutrition? A) Administration of PN requires clean technique. B) Central PN requires rapid dilution in a large volume of blood. C) Peripheral PN delivery is preferred over the use of a central line. D) Only water-soluble medications may be added to the PN by the nurse.

0.0/1.0 Correct Answer(s): B Central PN is hypertonic and requires rapid dilution in a large volume of blood. Because PN is an excellent medium for microbial growth, aseptic technique is necessary during administration. Administration through a central line is preferred over the use of peripheral PN and no medications may be added to PN by the nurse. Points Earned:

1.A 54-year-old patient admitted with type 2 diabetes, asks the nurse what "type 2" means. Which of the following is the most appropriate response by the nurse? A) "With type 2 diabetes, the body of the pancreas becomes inflamed." B) "With type 2 diabetes, insulin secretion is decreased and insulin resistance is increased." C) "With type 2 diabetes, the patient is totally dependent on an outside source of insulin." D) "With type 2 diabetes, the body produces autoantibodies that destroy b-cells in the pancreas."

0.0/1.0 Correct Answer(s): B In type 2 diabetes mellitus, the secretion of insulin by the pancreas is reduced and/or the cells of the body become resistant to insulin. Points Earned:

6.The nurse is beginning to teach a diabetic patient about vascular complications of diabetes. Which of the following information would be appropriate for the nurse to include? A) Macroangiopathy does not occur in type 1 diabetes but rather in type 2 diabetics who have severe disease. B) Microangiopathy is specific to diabetes and most commonly affects the capillary membranes of the eyes, kidneys, and skin. C) Renal damage resulting from changes in large- and medium-sized blood vessels can be prevented by careful glucose control. D) Macroangiopathy causes slowed gastric emptying and the sexual impotency experienced by a majority of patients with diabetes.

0.0/1.0 Correct Answer(s): B Microangiopathy occurs in diabetes mellitus. When it affects the eyes, it is called diabetic retinopathy. When kidneys are affected, the patient has nephropathy. When the skin is affected, it can lead to diabetic foot ulcers. Sexual impotency and slowed gastric emptying result from microangiopathy. Macroangiopathy can occur in either type 1 or type 2 diabetes. Points Earned:

12.The nurse has taught a patient admitted with diabetes, cellulitis, and osteomyelitis about the principles of foot care. The nurse evaluates that the patient understands the principles of foot care if the patient makes which of the following statements? A) "I should only walk barefoot in nice dry weather." B) "I should look at the condition of my feet every day." C) "I am lucky my shoes fit so nice and tight because they give me firm support." D) "When I am allowed up out of bed, I should check the shower water with my toes."

0.0/1.0 Correct Answer(s): B Patients with diabetes mellitus need to inspect the feet daily for broken areas that are at risk for delayed wound healing. Water temperature should be tested with the hands first. Properly fitted (not tight) shoes should be worn at all times. Points Earned:

4.A patient who has dysphagia as a consequence of a stroke is receiving enteral feedings through a percutaneous endoscopic gastrostomy (PEG). Which of the following interventions should the nurse integrate into this patient's care? A) Flush the tube with 30 ml of normal saline every 4 hours. B) Flush the tube before and after feedings if the patient's feedings are intermittent. C) Flush the PEG with 100 ml of sterile water before and after medication administration. D) To prevent fluid overload, avoid flushing when the patient is receiving continuous feeding.

0.0/1.0 Correct Answer(s): B The nurse should flush feeding tubes with 30 ml of water (not normal saline) every 4 hours during continuous feeding or before and after intermittent feeding. Flushes of 100 ml are excessive and may cause fluid overload in the patient. Points Earned:

6.The nurse has completed initial instruction with a patient regarding a weight loss program. The nurse determines that the teaching has been effective when the patient says A) "I plan to lose 4 lb a week until I have lost the 60-pound goal." B) "I will keep a diary of weekly weights to illustrate my weight loss." C) "I will restrict my carbohydrate intake to less than 30 g/day to maximize weight loss." D) "I should not exercise more than my program requires since increased activity increases the appetite."

0.0/1.0 Correct Answer(s): B The patient should monitor and record weight once per week. This prevents frustration at the normal variations in daily weights and may help the patient to maintain motivation to stay on the prescribed diet. Weight loss should occur at a rate of 1 to 2 lb/week. Exercise is a necessary component of any successful weight loss program. The diet should be well-balanced rather than lacking in specific components that may cause an initial weight loss but is not usually sustainable. Points Earned:

8.Which of the following assessments should the nurse prioritize in the care of a patient who has recently begun receiving parenteral nutrition (PN)? A) Skin integrity and bowel sounds B) Electrolyte levels and daily weights C) Auscultation of the chest and tests of blood coagulability D) Peripheral vascular assessment and level of consciousness (LOC)

0.0/1.0 Correct Answer(s): B The use of PN necessitates frequent and thorough assessments. Key focuses of these assessments include daily weights and close monitoring of electrolyte levels. Assessments of bowel sounds, integument, peripheral vascular system, LOC, chest sounds, and blood coagulation may be variously performed, but close monitoring of fluid and electrolyte balance supersedes these in importance. Points Earned:

3.Inspection of an elderly patient's mouth reveals the presence of white, curd-like lesions on the patient's tongue. The most likely etiology for this abnormal assessment finding is A) Herpesvirus. B) b. Candida albicans. C) Vitamin deficiency. D) Irritation from ill-fitting dentures.

0.0/1.0 Correct Answer(s): B White, curd-like lesions surrounded by erythematous mucosa are associated with oral candidiasis. Points Earned:

14.The patient received regular insulin 10 units subcutaneously at 8:30 pm for a blood glucose level of 253 mg/dl. The nurse plans to monitor this patient for signs of hypoglycemia at which of the following peak action times? A) 9:00 pm to 10:30 pm B) 10:30 pm to 11:30 pm C) 12:30 am to 1:30 am D) 2:30 am to 4:30 am

0.0/1.0 Correct Answer(s): B Regular insulin exerts peak action in 2 to 3 hours, making the patient most at risk for hypoglycemia between 10:30 pm and 11:30 pm. Points Earned:

5.In developing a weight reduction program with a 45-year-old patient who weighs 186 lb, the nurse encourages the patient to set a weight loss goal of how many pounds in 4 weeks? A) 1-2 B) 3-5 C) 4-8 D) 5-10

0.0/1.0 Correct Answer(s): C A realistic weight loss goal for patients is 1 to 2 lb/wk, which prevents the patient from becoming frustrated at not meeting weight loss goals. Points Earned:

1.When instructing a patient regarding a urine study for free cortisol, it is most important for the nurse to tell the patient to A) Save the first voided urine in the am. B) Maintain a high-sodium diet 3 days before collection. C) Try to avoid stressful situations during the collection period. D) Complete at least 30 minutes of strenuous exercise before collecting the urine sample.

0.0/1.0 Correct Answer(s): C A urine study for free cortisol requires a 24-hour urine collection. The patient should be instructed to avoid stressful situations and excessive physical exercise that could unduly increase cortisol levels. The patient should also maintain a low-sodium diet before and during the urine collection period. Points Earned:

4.In developing an effective weight reduction plan for an overweight patient who states a willingness to try to lose weight, it is most important for the nurse to first assess which of the following factors? A) The length of time the patient has been obese B) The patient's current level of physical activity C) The patient's social, emotional, and behavioral influences on obesity D) Anthropometric measurements, such as body mass index and skinfold thickness

0.0/1.0 Correct Answer(s): C Eating patterns are established early in life and eating has many meanings for people. To establish a weight reduction plan that will be successful for the patient, the nurse should first explore the social, emotional, and behavioral influences on the patient's eating patterns. Points Earned:

9.A patient is admitted with diabetes mellitus, has a glucose level of 380 mg/dl, and a moderate level of ketones in the urine. As the nurse assesses for signs of ketoacidosis, which of the following respiratory patterns would the nurse expect to find? A) Central apnea B) Hypoventilation C) Kussmaul respirations D) Cheyne-Stokes respirations

0.0/1.0 Correct Answer(s): C In diabetic ketoacidosis, the lungs try to compensate for the acidosis by blowing off volatile acids and carbon dioxide. This leads to a pattern of Kussmaul respirations, which are deep and nonlabored. Points Earned:

6.The nurse recognizes that the majority of patients' caloric needs should come from which of the following sources? A) Fats B) Proteins C) Polysaccharides D) Monosaccharides

0.0/1.0 Correct Answer(s): C It is imperative to ensure that an NG tube is situated in the GI tract rather than the patient's lungs. When an NG tube has been recently inserted, it is important to confirm this placement with an x-ray that will identify the tube's radiopaque tip. Points Earned:

5.The health care team is assessing a patient for acute pancreatitis after he presented to the emergency department with severe abdominal pain. Which of the following laboratory values are the best diagnostic indicators of acute pancreatitis? A) Gastric pH B) Blood glucose C) Serum amylase D) Serum potassium

0.0/1.0 Correct Answer(s): C Serum amylase levels indicate pancreatic function and are used to diagnose acute pancreatitis. Blood glucose, gastric pH, and potassium levels are not direct indicators of acute pancreatic dysfunction. Points Earned:

2.The nurse is caring for a patient admitted to the hospital for asthma who weighs 186 lb (84.5 kg). During dietary counseling, the patient asks the nurse how much protein he should ingest each day. How many grams of protein does the nurse recommend should be included in the diet based on the patient's current weight? A) 24 B) 41 C) 68 D) 93

0.0/1.0 Correct Answer(s): C The daily intake of protein should be between 0.8 and 1 g/kg of body weight. Thus this patient should take in between 68 and 84 g of protein per day in the diet. Points Earned:

4.The nurse is teaching a 54-year-old patient with diabetes about proper composition of the daily diet. The nurse explains that the guideline for carbohydrate intake is which of the following? A) 80% of daily intake B) Minimum of 80 g/day C) Minimum of 130 g/day D) Maximum of 130 g/day

0.0/1.0 Correct Answer(s): C The recommendation for carbohydrate intake is a minimum of 130 g/day. Low-carbohydrate diets are not recommended for diabetes management. Points Earned:

8.A 54-year-old patient with diabetes mellitus is scheduled for a fasting blood glucose level at 8:00 am. The nurse instructs the patient to only drink water after what time? A) 6:00 pm on the evening before the test B) 4:00 am on the day of the test C) Midnight before the test D) 7:00 am on the day of the test

0.0/1.0 Correct Answer(s): C Typically, a patient is ordered to be NPO for 8 hours before a fasting blood glucose level. For this reason, the patient who has a lab draw at 8:00 am should not have any food or beverages containing any calories after midnight. Points Earned:

3.The nurse is providing care for a 23-year-old woman who is a strict vegetarian. To prevent the consequences of iron deficiency, the nurse should recommend A) Brown rice and kidney beans. B) Cauliflower and egg substitutes. C) Soybeans and hot breakfast cereal. D) Whole-grain bread and citrus fruits.

0.0/1.0 Correct Answer(s): C Vegetarians are at a particular risk for iron deficiency, a problem that can be prevented by regularly consuming high-iron foods such as hot cereals and soybeans. The other foods listed are not classified as high sources of iron. Points Earned:

2. The nurse caring for a 54-year-old patient hospitalized with diabetes mellitus would look for which of the following laboratory test results to obtain information on the patient's past glucose control? A) Prealbumin level B) Urine ketone level C) Fasting glucose level D) Glycosylated hemoglobin level

0.0/1.0 Correct Answer(s): D A glycosylated hemoglobin level detects the amount of glucose that is bound to red blood cells (RBCs). When circulating glucose levels are high, glucose attaches to the RBCs and remains there for the life of the blood cell, which is approximately 120 days. Thus the test can give an indication of glycemic control over approximately 2 to 3 months.

5.A patient's recent medical history is indicative of diabetes insipidus. The nurse would perform patient education related to which of the following diagnostic tests? A) Thyroid scan B) Fasting glucose test C) Oral glucose tolerance D) Water deprivation test

0.0/1.0 Correct Answer(s): D A water deprivation test is used to diagnose the polyuria that accompanies diabetes insipidus. Glucose tests and thyroid tests are not directly related to the diagnosis of diabetes insipidus. Points Earned:

6.The nurse is performing a focused abdominal assessment of a patient who has been recently admitted. In order to palpate the patient's liver, the nurse will palpate the patient's A) Left lower quadrant. B) Left upper quadrant. C) Right lower quadrant. D) Right upper quadrant. Feedback: INCORRECT Although the left lobe of the liver is located in the left upper quadrant of the abdomen, the bulk of the liver is located in the right upper quadrant. Points Earned:

0.0/1.0 Correct Answer(s): D Although the left lobe of the liver is located in the left upper quadrant of the abdomen, the bulk of the liver is located in the right upper quadrant. Points Earned:

13.A patient is admitted with diabetes mellitus, malnutrition, and cellulitis. The patient's potassium level is 5.6 mEq/L. The nurse considers that which of the following could be a contributing factor for this lab result (select all that apply)? A) The level may be increased as a result of dehydration that accompanies hyperglycemia. B) The patient may be excreting extra sodium and retaining potassium because of malnutrition. C) The level is consistent with renal insufficiency that can develop with renal nephropathy. D) The level may be raised as a result of metabolic ketoacidosis caused by hyperglycemia.

0.0/3.0 Correct Answer(s): A, C, D Malnutrition does not cause sodium excretion accompanied by potassium retention; thus it is not a contributing factor to this patient's potassium level. The additional stress of cellulitis may lead to an increase in the patient's serum glucose levels. Dehydration may cause hemoconcentration, resulting in elevated serum readings. Kidneys may have difficulty excreting potassium if renal insufficiency exists. Finally, the nurse must consider the potential for metabolic ketoacidosis since potassium will leave the cell when hydrogen enters in an attempt to compensate for a low pH. Points Earned:

7.A community health nurse is conducting an initial assessment of a new patient. Which of the following assessments would the nurse include when screening the patient for metabolic syndrome (select all that apply)? A) Blood pressure B) Resting heart rate C) Physical endurance D) Waist circumference E) Fasting blood glucose

0.0/3.0 Correct Answer(s): A, D, E The diagnostic criteria for metabolic syndrome include blood pressure, fasting blood glucose, waist circumference, triglycerides, and HDL cholesterol. Points Earned:

4.The nurse would recognize that the liver performs which of the following functions (select all that apply)? A) Bile storage B) Detoxification C) Red blood cell (RBC) destruction D) Protein metabolism E) Steroid metabolism

0.0/4.0 Correct Answer(s): B, C, D, E The liver performs multiple major functions that aid in the maintenance of homeostasis. These include metabolism of proteins and steroids as well as detoxification of drugs and metabolic waste products. The liver produces bile, but storage occurs in the gall bladder. The Kupffer cells of the liver participate in the breakdown of old RBCs. Points Earned:

84-year-old man is being discharged from the hospital. He was admitted with complications from esophageal cancer. PEG tube has been placed to allow tube feedings. He lives with his wife, who is 82 years old. Has no other family in town 1.What risks are associated with a PEG that you should discuss with him and his wife? 2.What skills must he and his wife learn before the time of discharge? 3.How should his response to treatment be managed?

1. Aspiration, ensuring proper use of formula 2. Proper tube care 3. His weight and hydration status

A young woman calls a triage hotline complaining of severe nausea for the past 6 hours. She denies any emesis, but is unable to obtain any relief from her nausea. She states her temperature is "a little higher than normal" at 99.5o F. She has had no recent illnesses. 1.What other information should you obtain from her? 2.What remedies could she use to relieve her nausea? 3.If she started vomiting, would you change your advice?

1. Dietary history (and if anyone else has nausea), history of nausea, and discussion of possible pregnancy 2. Breathing exercises and peppermint tea 3. No, unless her vomiting is unremitting or includes blood.

27-year-old woman comes into the clinic with fatigue. She has lost 10 pounds in the past 2 months. She is a graduate student working on her dissertation. History of Crohn's disease Claims she is unable to watch her diet because she "doesn't have time to think about that" In addition to fatigue, she reports having diarrhea and no appetite. 1.What nutrients is she likely deficient in? 2.What dietary recommendations can be made? 3.How can you help her monitor her diet without added stress?

1. Fluids, electrolytes, fat 2. She should begin a high-fat, high-protein diet. 3. Provide her with a list of foods that she should try to eat.

76-year-old man complains of feeling like food is "stuck" in his esophagus. Also complains of experiencing epigastric pain while eating History of smoking 1 pack per day for 40 years and hypertension EGD and biopsy samples taken reveal esophageal cancer. Scheduled for surgical removal of affected area 1.What alterations in function is he likely to have after surgery? 2.What complications might he experience with food intake? 3.What patient teaching should you perform?

1. He will have an NG tube immediately after surgery, and then he will need to maintain an upright position for at least 2 hours after eating. 2. He may need a permanent feeding gastrostomy. 3. Any changes in eating habits should be discussed, along with proper care of wound/gastrostomy teaching.

21-year-old woman comes into her physician's office for her routine check-up. She states that she has switched to a vegetarian diet because she is concerned with cruelty to animals. She eats mainly fast food and salads from her college cafeteria. She consumes milk, eggs, and cheese. No abnormalities are noted on assessment. 1.What deficiency might her diet include? 2.What foods would you stress to compensate for the lack of meat? 3.What resources could you recommend to help her learn more about a vegetarian's needs?

1. Iron deficiency 2. Beans, legumes, soy products 3. The USDA website

40-year-old woman was previously admitted to the hospital for exacerbation of Cr0hn's disease. After beginning high-dose IV corticosteroid treatment for Crohn's disease, she develops massive upper GI bleeding. Bright red blood hematemesis and melena She is treated with iced saline NG irrigations and blood transfusions. The bleeding is stopped. The following day, she has a recurrence of bleeding through her nasogastric tube and bright blood from her rectum. Gastroscopy reveals a peptic ulcer. After a third day of more emesis of blood clots, she is taken to surgery, where a Billroth II is performed. 1.What is the likely cause of the massive gastrointestinal bleeding? 2.Following the surgery, what is the priority nursing intervention for her? 3.What nutritional changes should she make because of the diagnosis of peptic ulcer? 4.What patient teaching should you do with her?

1. Peptic ulcer 2. Closely monitor I & O and vital signs. 3. Avoid acidic foods, caffeine, and alcohol. 4. Nutritional changes, need for taking only prescription medications, how to avoid future incidences, avoiding smoking, importance of monitoring for bleeding, etc.

20-year-old man complains of loss of appetite and occasional lower sternal chest pain 30 to 60 minutes after meals. He claims symptoms began about 6 months ago. He has a history of asthma. States he has needed inhaler "more than usual" Does not know what makes it worse or better EGD suggests GERD. 1.How may factors be affecting his GERD? 2.What nutritional counseling should you do? 3.He states that he doesn't want to take any medication. How can you best advise him? 4.What are long-term complications of GERD?

1. Respiratory history 2. Avoid acids foods, alcohol, and caffeine. 3. Discuss with him that if untreated, GERD can lead to esophagitis and Barrett's esophagus. 4. Esophagitis, Barrett's esophagus, respiratory problems, and dental erosion

75-year-old woman is scheduled to have exploratory surgery tomorrow for a bowel obstruction. History of type 2 diabetes Orders call for a PICC insertion because the physician anticipates a need for resting her GI tract. Her son is here for the surgery, and he is concerned about when she can eat "regular food" after the surgery. She lives in an assisted living facility, and her son visits a few times a week. 1.What can you tell the son about feeding after her surgery? 2.What are you concerned about because she is a diabetic? 3.What will you teach her and her son to monitor? 4.What resources are available to help her at home?

1. She will be fed through the PICC only until she is able to eat 60% of her calories orally. Then she will be back to eating "normally." 2. Her blood glucose should be monitored closely. A slight increase in blood glucose is to be expected at the outset of PN. 3. If she continues PN at home, she should be taught proper tube care and should learn the signs of infection. 4. 4. OLEY foundation (see resources at end of chapter for a complete list)

45-year-old woman is hospitalized for shortness of breath and respiratory distress. Physical examination findings Blood pressure 150/72 Heart rate 104 Respiratory rate 30 Temperature 98.3° F SaO2 88% Height 5'5" Weight 320 pounds History of hypertension, type 2 diabetes, COPD, obesity She states that she's "tired of being like this." 1.What opportunities for education and support do you have? 2.What other problems is she likely to have related to her weight? 3.What treatment options are available for her? 4.What tools may help her with behavior modification? 5.If she wants to have bariatric surgery, what risks does the surgery pose?

1. Teach her the negative effects of obesity. 2. Pain in joints, high cholesterol, sleep apnea 3. Drug therapy for weight reduction, such as Alli, or bariatric surgery 4. Support groups or individual therapy sessions 5. Postanesthesia problems due to her weight and pain following surgery

A patient is jaundiced and her stools are clay colored (gray). This is most likely related to: a. decreased bile flow into the intestine b. increased production of urobilinogen c. increased production of cholecystokinin d. increased bile and bilirubin in the blood

A

A patient with a history of peptic ulcer disease has presented to the emergency department with complaints of severe abdominal pain and a rigid, boardlike abdomen, prompting the health care team to suspect a perforated ulcer. Which of the following actions should the nurse anticipate? A) a. Providing IV fluids and inserting a nasogastric tube B) b. Administering oral bicarbonate and testing the patient's gastric pH level C) c. Performing a fecal occult blood test and administering IV calcium gluconate D) d. Starting parenteral nutrition and placing the patient in a high-Fowler's position

A

After administering a dose of promethazine (Phenergan) to a patient with nausea and vomiting, the nurse explains that which of the following may be experienced as a common temporary adverse effect of the medication? A) a. Drowsiness B) b. Reduced hearing C) c. Sensation of falling D) d. Photosensitivity

A

During the initial postoperative period following bariatric surgery, the nurse recognizes the importance of monitoring obese patients for respiratory insufficiency based on knowledge that A) a. The body stores anesthetics in adipose tissue. B) b. Intubation may be difficult because of extra chin skin folds. C) c. Postoperative pain may cause a decreased respiratory rate. D) d. The patient's head must remain flat for a minimum of 2 hours postprocedure.

A

The nurse confirms initial placement of a blindly inserted small bore NG feeding tube by: a. x-ray b. air insufflation c. observing patient for coughing d. pH measurement of gastric aspirate

A

The nurse is caring for a patient who is 5'6" tall and weighs 186 lb. The nurse has discussed reasonable weight loss goals and a low-calorie diet with the patient. Which of these statements made by the patient indicates a need for further teaching? A) a. "I will limit intake to 500 calories a day." B) b. "I will try to eat very slowly during mealtimes." C) c. "I'll try to pick foods from all of the basic food groups." D) d. "It's important for me to begin a regular exercise program."

A

The percentage of daily calories for a healthy individual consists of: a. 50% carbohydrates, 25% protein, 25% fat, and <10% of fat from saturated fatty acids b. 65% carbohydrates, 25% protein, 25% fat, and >10% of fat from saturated fatty acids c. 50% carbohydrates, 40% protein, 10% fat, and <10% of fat from saturated fatty acids d. 40% carbohydrates, 30% protein, 30% fat, and >10% of fat from saturated fatty acids

A

When administered a dose of metoclopramide (Reglan), a patient complains of nausea. The nurse would teach the patient to report which of the following potential adverse effects? A) a. Tremors B) b. Constipation C) c. Double vision D) d. Numbness in the fingers and dose

A

When assessing a patient's abdomen, it would be most appropriate to A) a. Auscultate the abdomen before palpation. B) b. Palpate the abdomen before auscultation. C) c. Percuss the abdomen before auscultation. D) d. Perform deep palpation before light palpation.

A

When preparing a patient for a capsule endoscopy study, the nurse should A) a. Ensure the patient understands the required bowel preparation. B) b. Have the patient return to the procedure room for removal of the capsule. C) c. Teach the patient to maintain a clear liquid diet throughout the procedure. D) d. Explain to the patient that conscious sedation will be used during placement of the capsule.

A

Which of the following methods should be used to confirm the placement of a patient's newly-inserted nasogastric (NG) feeding tube? A) a. X-ray B) b. Aspiration C) c. Air auscultation D) d. Measurement of external length

A

After providing discharge instructions to a patient following a laparoscopic cholecystectomy, the nurse recognizes that teaching has been effective when the patient states, A) "I can remove the bandages on my incisions tomorrow and take a shower." B) "I can expect some yellow-green drainage from the incision for a few days." C) "I should plan to limit my activities and not return to work for 4 to 6 weeks." D) "I will always need to maintain a low-fat diet since I no longer have a gallbladder."

A) "I can remove the bandages on my incisions tomorrow and take a shower." Feedback: After a laparoscopic cholecystectomy, the patient will have Band-Aids in place over the incisions. Patients are discharged the same (or next) day and have few restrictions on activities of daily living. Drainage from the incisions would be abnormal, and the patient should be instructed to call the health care provider if this occurs. A low-fat diet may be recommended for a few weeks after surgery but will not be a life-long requirement.

During change-of-shift report, the nurse learns about the following four patients. Which patient requires the most rapid assessment? A) 45-year-old with cirrhosis and severe ascites who has an oral temperature of 102° F (38.8° C) B) 48-year-old who has compensated cirrhosis and is complaining of anorexia C) 50-year-old with chronic pancreatitis who has gnawing abdominal pain D) 56-year-old who is recovering from a laparoscopic cholecystectomy and has severe shoulder pain

A) 45-year-old with cirrhosis and severe ascites who has an oral temperature of 102° F (38.8° C) Feedback: This patient's history and fever suggest spontaneous bacterial peritonitis, which would require rapid assessment and interventions such as antibiotic therapy. The clinical manifestations for the other patients are consistent with their diagnoses and do not indicate complications are occurring.

The nurse identifies that which of the following patients is at highest risk for developing colon cancer? A) A 32-year-old female with a 12-year history of ulcerative colitis B) A 52-year-old male who has followed a vegetarian diet for 24 years C) A 28-year-old male who has a body mass index of 27 kg/m2 D) A 58-year-old female who takes prescribed estrogen replacement therapy

A) A 32-year-old female with a 12-year history of ulcerative colitis Feedback: Rationale: Risk for colon cancer include personal history of inflammatory bowel disease (especially ulcerative colitis for longer than 10 years); obesity (body mass index =30 kg/m2); family (first-degree relative) or personal history of colorectal cancer, adenomatous polyposis, hereditary nonpolyposis colorectal cancer syndrome; red meat ( =7 servings/week); cigarette use; and alcohol ( =4 drinks/week).

A patient who has been NPO during treatment for nausea and vomiting caused by gastric irritation is to start oral intake. Which of these should the nurse offer to the patient? A) A dish of lemon gelatin B) A cup of coffee with cream C) A bowl of hot chicken broth D) A glass of orange juice

A) A dish of lemon gelatin Feedback: Clear liquids are usually the first foods started after a patient has been nauseated. Acidic foods such as orange juice, very hot foods, and coffee are poorly tolerated when patients have been nauseated.

When caring for a patient with liver disease, the nurse recognizes the need to prevent bleeding resulting from altered clotting factors and rupture of varices. Which of the following nursing interventions would be appropriate to achieve this outcome (select all that apply)? A) Advise patient to use soft-bristle toothbrush and avoid ingestion of irritating food. B) Teach patient to avoid straining at stool, vigorous blowing of nose, and coughing. C) Use smallest gauge possible when giving injections or drawing blood. D) Apply gentle pressure for the shortest possible time period after performing venipuncture. E) Instruct patient to avoid aspirin and NSAIDs to prevent hemorrhage when varices are present.

A) Advise patient to use soft-bristle toothbrush and avoid ingestion of irritating food. B) Teach patient to avoid straining at stool, vigorous blowing of nose, and coughing. C) Use smallest gauge possible when giving injections or drawing blood. E) Instruct patient to avoid aspirin and NSAIDs to prevent hemorrhage when varices are present. Feedback: Using the smallest gauge for injections will minimize the risk of bleeding into the tissues. Avoiding straining, nose blowing, and coughing will reduce the risk of hemorrhage at these sites. The use of a soft-bristle toothbrush and avoidance of irritating food will reduce injury to highly vascular mucous membranes. The nurse should apply gentle but prolonged pressure to venipuncture sites to minimize the risk of bleeding.

A patient with cirrhosis has 4+ pitting edema of the feet and legs. The data indicate that it is most important for the nurse to monitor the patient's A) Albumin level. B) Temperature. C) Activity level. D) Hemoglobin

A) Albumin level. Feedback: The low oncotic pressure caused by hypoalbuminemia is a major pathophysiologic factor in the development of edema. The other parameters also should be monitored, but they are not directly associated with the patient's current symptoms.

A patient is admitted to the hospital for evaluation of right lower quadrant abdominal pain with nausea and vomiting. Which action should the nurse take? A) Apply an ice pack to the right lower quadrant. B) Encourage the patient to take sips of clear liquids. C) Check for rebound tenderness. D) Assist the patient to cough and deep breathe.

A) Apply an ice pack to the right lower quadrant. Feedback: The patient's clinical manifestations are consistent with appendicitis, and application of an ice pack will decrease inflammation at the area. The patient should be NPO in case immediate surgery is needed. Checking for rebound tenderness frequently is unnecessary and uncomfortable for the patient. The patient will need to know how to cough and deep breathe postoperatively, but coughing will increase pain at this time.

A patient is admitted with anorexia nervosa and a serum potassium level of 2.4 mEq/L. What is most important for the nurse to observe for in this patient? A) Cardiac dysrhythmias B) Muscle weakness C) Increased urine output D) Anemia and leukopenia

A) Cardiac dysrhythmias Feedback: Rationale: A serum potassium level less than 2.5 mEq/L indicates severe hypokalemia; this can lead to life-threatening cardiac dysrhythmias (e.g., bradycardia, tachycardia, or ventricular dysrhythmias). Other manifestations of potassium deficiency include muscle weakness and and renal failure. Patients with anorexia nervosa commonly have iron-deficiency anemia and an elevated blood urea nitrogen level that is related to intravascular volume depletion and abnormal renal function.

The health care provider orders lactulose for a patient with hepatic encephalopathy. The nurse will monitor for effectiveness of this medication for this patient by assessing which of the following? A) Decreased ammonia levels B) Relief of abdominal pain C) Decreased liver enzymes D) Relief of constipation

A) Decreased ammonia levels Feedback: Hepatic encephalopathy is a complication of liver disease and is associated with elevated serum ammonia levels. Lactulose traps ammonia in the intestinal tract. Its laxative effect then expels the ammonia from the colon, resulting in decreased serum ammonia levels and correction of hepatic encephalopathy.

When the nurse is caring for a patient with acute pancreatitis, which assessment finding is of most concern? A) Palpable abdominal mass B) Absent bowel sounds C) Abdominal tenderness D) Left upper quadrant pain

A) Palpable abdominal mass Feedback: A palpable abdominal mass may indicate the presence of a pancreatic abscess, which will require rapid surgical drainage to prevent sepsis. Absent bowel sounds, abdominal tenderness, and left upper quadrant pain are common in acute pancreatitis and do not require rapid action to prevent further complications.

A patient who is hospitalized with watery, incontinent diarrhea is diagnosed with Clostridium difficile. Which action will the nurse include in the plan of care? A) Place the patient in a private room with contact isolation. B) Teach the patient about why antibiotics are not being used. C) Order a diet with no dairy products for the patient. D) Educate the patient about proper food handling and storage.

A) Place the patient in a private room with contact isolation. Feedback: Because C. difficile is highly contagious, the patient should be placed in a private room and contact precautions should be used. There is no need to restrict dairy products for this type of diarrhea. Metronidazole (Flagyl) is frequently used to treat C. difficile. Improper food handling and storage do not cause C. difficile.

A patient hospitalized with an acute exacerbation ofulcerative colitis is having 14 to 16 bloody stools a day and crampy abdominal pain associated with the diarrhea. The nurse will plan to A) Place the patient on NPO status. B) Administer cobalamin (vitamin B12) injections. C) Administer IV metoclopramide (Reglan). D) Teach the patient about total colectomy surgery.

A) Place the patient on NPO status. Feedback: An initial therapy for an acute exacerbation of inflammatory boweldisease (IBD) is to rest the bowel by making the patient NPO. Cobalamin (vitamin B12) is absorbed in the ileum, which is not affected by ulcerative colitis. Although total colectomy is needed for some patients, there is no indication that this patient is a candidate. Metoclopramide increases peristalsis and will worsen symptoms.

A hospitalized patient who has been taking antibiotics for several days develops watery diarrhea. Which action should the nurse take first? A) Place the patient on contact precautions. B) Provide education about handwashing. C) Notify the health care provider. D) Obtain a stool specimen for analysis.

A) Place the patient on contact precautions. Feedback: The patient's history and new onset diarrhea suggest a C. difficile infection, which requires implementation of contact precautions to prevent spread of the infection to other patients. The other actions also are appropriate but can be accomplished after contact precautions are implemented.

A patient who has advanced cirrhosis is receiving lactulose (Cephulac). Which finding by the nurse indicates that the medication is effective? A) The patient is alert and oriented. B) The patient's bilirubin level decreases. C) The patient has at least one stool daily. D) The patient denies nausea or anorexia.

A) The patient is alert and oriented. Feedback: The purpose for lactulose in the patient with cirrhosis is to lower ammonia levels and prevent encephalopathy. Although lactulose may be used to treat constipation, that is not the purpose for this patient. Lactulose will not decrease nausea and vomiting or lower bilirubin levels.

Health risks associated with obesity include (select all that apply) a. colorectal cancer b. rheumatoid arthritis c. polycystic ovary disease d. nonalcoholic steatohepatitis e. systemic lupus erythematosus

A, C, D

Which of the following criteria are needed for a diagnosis of metabolic syndrome (select all that apply) a. hypertension b. elevated triglycerides c. elevated plasma glucose d. increased waist circumference e. decreased low density lipoproteins

A,B,C,D

A normal physical assessment finding of the GI system is/are: (select all the apply) a. nonpalpable liver and spleen b. borborygmi in upper right quadrant c. tympany on percussion of the abdomen d. liver edge 2 to 4 cm below the costal margin e. finding a firm nodular edge on the rectal exam

A,C

A community health nurse is conducting an initial assessment of a new patient. Which of the following assessments would the nurse include when screening the patient for metabolic syndrome (select all that apply)? A) a. Blood pressure B) b. Resting heart rate C) c. Physical endurance D) d. Waist circumference E) e. Fasting blood glucose

A,D,E

Diagnostic testing is planned for a patient with a suspected peptic ulcer. The nurse explains to the patient that the most reliable test for determining the presence and location of an ulcer is a(n) 1.endoscopy. 2.gastric analysis. 3.barium swallow. 4.serologic test for Helicobacter pylori.

Answer: 1 Rationale: Endoscopy is the primary tool for diagnosing the source of upper gastrointestinal bleeding.

The health care provider orders a 10% fat emulsion solution to be administered to a critically ill patient who is currently receiving peripheral parenteral nutrition. Which of the following assessment findings would alert the nurse to a systemic problem related to lipid administration? 1.The onset of vomiting and fever 2.Retention of fluid with peripheral edema 3.A random capillary blood glucose level of 148 mg/dl 4.Erythema, tenderness, and exudate at the catheter insertion site

Answer: 1 Rationale: Patients receiving lipids are at extreme risk for infection because fat emulsions at room temperature are a medium for microorganism growth. Catheter-related infections can occur in patients receiving parenteral nutrition; local or systemic manifestations of infection may occur. Adverse reactions from lipid administration include allergic reactions, dyspnea, cyanosis, fever, flushing, phlebitis, chest and back pain, and pain at the IV site. Hyperglycemia and fluid overload are other potential complications; however, infection would be the highest risk for patients receiving lipids.

Which of the following patients is it most important for the nurse to refer to the dietitian for a complete nutritional assessment? 1.A 38-year-old diabetic patient who is undergoing laser eye surgery 2.A 55-year-old with a history of alcoholism who is hospitalized with a fractured femur resulting from a fall 3.A 24-year-old who has been taking a burst of corticosteroid therapy for 1 week for treatment of an asthma exacerbation 4.A 45-year-old hospitalized with nausea and abdominal pain who has had no oral intake and has received only IV fluids of D5½NS for 6 days

Answer: 2 Rationale: An individual who is at nutritional risk should have a full nutritional assessment. The individual at greatest risk is the individual with chronic alcoholism and a fractured femur. Drugs such as corticosteroids place the individual at risk for malnutrition, but the person in option 3 is taking the medication only on a short-term basis. Patients with chronic illnesses (such as diabetes) (option 1) are at nutritional risk but not as high risk as option 2. The person in option 4 is at risk for malnutrition after 10 days of receiving only intravenous fluids.

When assessing the patient's nutritional status, the nurse asks which drugs the patient takes primarily because 1.foods alter the absorption or bioavailability of every drug. 2.if the patient skips a meal, medications may not be taken. 3.some drugs increase the requirements for essential nutrients. 4.medications should be taken with food to prevent GI irritation.

Answer: 3 Rationale: Certain drugs (such as folic acid, riboflavin, and fat-soluble vitamins) may increase nutrient requirements. Some medications can be taken with food; other medications must be taken without food. Food may alter the absorption or bioavailability of some drugs, but not all drugs.

Following discharge instructions for a patient who has had bariatric surgery for treatment of obesity, the nurse determines that additional teaching is needed when the patient says, 1. "I shouldn't eat concentrated sweets." 2. "I can eat small, frequent meals throughout the day." 3. "I should drink several glasses of fluids with my meals." 4. "I will need to have a cobalamin injection once a month."

Answer: 3 Rationale: Discharge teaching for a patient after bariatric surgery may include six small meals/day, diet high in protein and low in fat and carbohydrates, avoidance of ingestion of solids with fluids, avoidance of large amounts of fluids at one time, fluid intake possibly restricted to less than 1000 ml/day, and avoidance of sugary foods. The dietary restrictions will help to prevent dumping syndrome and will aid in weight loss. Cobalamin injections or intranasal spray will prevent cobalamin deficiency anemia.

After the nurse teaches a patient with gastroesophageal reflux disease (GERD) about recommended dietary modifications, which statement by the patient indicates that the teaching has been effective? 1."I can have a glass of low-fat milk at bedtime." 2."I will have to eliminate all spicy foods from my diet." 3."I will have to use herbal teas instead of caffeinated drinks." 4."I should keep something in my stomach all the time to neutralize the excess acids."

Answer: 3 Rationale: Patients with gastroesophageal reflux disease should avoid foods (such as tea and coffee) that decrease lower esophageal pressure. Patients should also avoid milk, especially at bedtime, as it increases gastric acid secretion. Patients may eat spicy foods unless these foods cause reflux. Small, frequent meals help prevent overdistention of the stomach, but patients should avoid late evening meals and nocturnal snacking.

A patient with persistent vomiting of 3 days' duration is seen at the urgent care center because of increasing weakness. Intravenous therapy with lactated Ringer's solution is started, and arterial blood gases (ABGs) are ordered. Which of the following ABG results would the nurse expect? 1.pH 7.4; PaCO2 40 mm Hg; HCO3- 25 mEq/L 2.pH 7.3; PaCO2 50 mm Hg; HCO3- 20 mEq/L 3.pH 7.6; PaCO2 30 mm Hg; HCO3- 40 mEq/L 4.pH 7.48; PaCO2 40 mm Hg; HCO3- 30 mEq/L

Answer: 3 Rationale: Vomiting is a cause of metabolic alkalosis; the arterial blood gases indicate partially compensated metabolic alkalosis. The pH is greater than 7.45 (alkalosis); the HCO3- is above 26 mEq/L (metabolic); and the PaCO2 is less than 35 mm Hg (partially compensated).

The nurse determines that teaching regarding a weight-loss program has been effective when the patient says, 1. "I will keep a diary of daily weights to chart my weight loss." 2. "I plan to lose 4 pounds a week until I have lost my goal of 60 pounds." 3. "I should not exercise more than what is required because increased activity increases the appetite." 4. "I plan to join a behavior-modification group to make permanent changes necessary for weight control."

Answer: 4 Rationale: Behavior-modification programs deemphasize the diet and focus on how and when to eat; support groups offer support and information on dieting tips. Patients should set a weight loss goal of 1 to 2 pounds per week. Weight should be checked weekly; daily weights are not recommended because of the frequent fluctuations resulting from retained water (including urine) and elimination of feces. No evidence indicates that increased activity promotes an increase in appetite or leads to dietary excess.

When teaching a patient with a history of upper GI bleeding to check the stools for blood, the nurse informs the patient that 1.if vomiting of bright red blood occurs, stools will not be black and sticky. 2.blood is never obvious in stools and must be detected by fecal occult blood testing. 3.acute bleeding in the upper GI tract will result in bright red blood in the stools. 4.stools that are black and tarry occur with prolonged bleeding from the stomach or small intestine.

Answer: 4 Rationale: Melena (black, tarry stools) indicates slow bleeding from an upper gastrointestinal source. The longer the passage of blood through the intestines, the darker the stool color, because of the breakdown of hemoglobin and the release of iron.

An important factor associated with both short-term and long-term weight-loss success is 1. higher initial body mass index. 2. simultaneous smoking cessation. 3. fewer dieting attempts in the past year. 4. a strong desire to improve appearance.

Answer: 4 Rationale: Motivation to lose weight is essential for a favorable and successful outcome.

A patient has an elevated blood level of indirect (unconjugated) bilirubin. Onecause of this finding is that: a. the gallbladder is unable to contract to release stored bile b. bilirubin is not being conjugated and excreted into the bile by the liver c. the Kupffer cells in the liver are unable to remove bilirubin from the blood d. there is an obstruction in the biliary tract preventing flow of bile into the small intestine

B

A patient is receiving peripheral parenteral nutrition. The parenteral nutrition solution is completed before the new solution arrives on the unit. The nurse administers: a. 20% intralipids b. 5% dextrose solution c. 5% Ringer's lactate solution d. 0.45% normal saline solution

B

A patient who has dysphagia as a consequence of a stroke is receiving enteral feedings through a percutaneous endoscopic gastrostomy (PEG). Which of the following interventions should the nurse integrate into this patient's care? A) a. Flush the tube with 30 ml of normal saline every 4 hours. B) b. Flush the tube before and after feedings if the patient's feedings are intermittent. C) c. Flush the PEG with 100 ml of sterile water before and after medication administration. D) d. To prevent fluid overload, avoid flushing when the patient is receiving continuous feeding.

B

A patient who has suffered severe burns in a motor vehicle accident will soon be started on parenteral nutrition (PN). Which of the following principles should guide the nurse's administration of the patient's nutrition? A) a. Administration of PN requires clean technique. B) b. Central PN requires rapid dilution in a large volume of blood. C) c. Peripheral PN delivery is preferred over the use of a central line. D) d. Only water-soluble medications may be added to the PN by the nurse.

B

An 80-year-old man states that although he adds a lot of salt to his food it still does not have much taste. The nurse's response is based on the knowledge that the older adult: a. should not experience changes in taste b. has a loss of taste buds, especially sweet and salt c. has some loss of taste but no difficulty chewing food d. loses the sense of taste because the ability to smell is decreased

B

As gastric contents move into the small intestine, the bowel is normally protected from the acidity of gastric contents by the: a. inhibition of secretin release b. release of bicarbonate by the pancreas c. release of pancreatic digestive enzymes d. release of gastrin by the duodenal mucosa

B

During an examination of the abdomen the nurse should: a. position the patient in the supine position with the bed flat and the knees straight b. listen in the epigastrium and all found quadrants for 2 to 5 minutes for bowel sounds c. use the following order of techniques: inspections, palpation, percussion, auscultation d. describe bowel sounds if no sound is heard in the lower right quadrant after two minutes

B

During starvation, the order in which the body obtains substrate for energy is: a. visceral protein, skeletal protein, fat, glycogen b. glycogen, skeletal protein, fat stores, visceral protein c. visceral protein, fat stores, glycogen, skeletal protein d. fat stores, skeletal protein, visceral protein, glycogen

B

In preparing a patient for a colonoscopy, the nurse explains that: a. a signed permit is not necessary b. sedation may be used during the procedure c. only one cleansing enema is necessary for preparation d. a light meal should be eaten the day before the procedure

B

Inspection of an elderly patient's mouth reveals the presence of white, curd-like lesions on the patient's tongue. The most likely etiology for this abnormal assessment finding is A) a. Herpesvirus. B) b. Candida albicans. C) c. Vitamin deficiency. D) d. Irritation from ill-fitting dentures.

B

M.J. calls to tell the nurse that her elderly mother, who is 85 years of age, has been nauseated all day and has vomited twice. Before the nurse hangs up and telephones the health care provider to communicate your assessment data, the nurse should instruct M.J. to: a.administer antispasmodic drugs and observe skin turgor b. give her mother sips of water and elevate the head of her bed to prevent aspiration c. offer her mother a high-protein liquid supplement to drink to maintain her nutritional needs d. offer her mother large quantities of gatorade to drink because elderly people are at risk for sodium depletion

B

Several patients are seen at an urgent care center with symptoms of nausea, vomiting, and diarrhea that began 2 hours ago while attending a large family reunion potluck dinner. You question the patient specifically about foods they ingested containing a. beef b. meat and milk c. poultry and eggs d. home-preserved vegetables

B

The nurse has completed initial instruction with a patient regarding a weight loss program. The nurse determines that the teaching has been effective when the patient says A) a. "I plan to lose 4 lb a week until I have lost the 60-pound goal." B) b. "I will keep a diary of weekly weights to illustrate my weight loss." C) c. "I will restrict my carbohydrate intake to less than 30 g/day to maximize weight loss." D) d. "I should not exercise more than my program requires since increased activity increases the appetite."

B

The nurse is caring for a 45-year-old woman with a herniated lumbar disk and obesity. The patient realizes that weight loss is necessary to lessen back strain. The patient is 5'6" tall and weighs 186 lb (84.5 kg) with a body mass index (BMI) of 28 kg/m2. The nurse explains to the patient that this measurement places her in which of the following weight categories? A) a. Normal weight B) b. Overweight C) c. Obese D) d. Morbidly obese

B

The nurse is caring for a patient treated with intravenous fluid therapy for severe vomiting. As the patient recovers and begins to tolerate oral intake, the nurse understands that which of the following food choices would be most appropriate? A) a. Ice tea B) b. Dry toast C) c. Warm broth D) d. Plain hamburger

B

The patient who is admitted with a diagnosis of diverticulitis and a history of irritable bowel disease and gastroesophageal reflux disease (GERD) has received a dose of Mylanta 30 ml PO. The nurse would evaluate its effectiveness by questioning the patient as to whether which of the following symptoms has been resolved? A) a. Diarrhea B) b. Heartburn C) c. Constipation D) d. Lower abdominal pain

B

The results of a patient's recent endoscopy indicate the presence of peptic ulcer disease (PUD). Which of the following teaching points should the nurse provide to the patient in light of his new diagnosis? A) a. "You'll need to drink at least two to three glasses of milk daily." B) b. "It would likely be beneficial for you to eliminate drinking alcohol." C) c. "Many people find that a minced or pureed diet eases their symptoms of PUD." D) d. "Your medications should allow you to maintain your present diet while minimizing symptoms."

B

Which of the following assessments should the nurse prioritize in the care of a patient who has recently begun receiving parenteral nutrition (PN)? A) a. Skin integrity and bowel sounds B) b. Electrolyte levels and daily weights C) c. Auscultation of the chest and tests of blood coagulability D) d. Peripheral vascular assessment and level of consciousness (LOC)

B

The family of a patient newly diagnosed with hepatitis A asks the nurse what they can do to prevent becoming ill themselves. Which of the following responses by the nurse is most appropriate? A) "The hepatitis vaccine will provide immunity from this exposure and future exposures." B) "An injection of immunoglobulin will need to be given to prevent or minimize the effects from this exposure." C) "You will need to be tested first to make sure you don't have the virus before we can treat you." D) "I am afraid there is nothing you can do since the patient was infectious before admission."

B) "An injection of immunoglobulin will need to be given to prevent or minimize the effects from this exposure." Feedback: Immunoglobulin provides temporary (1-2 months) passive immunity and is effective for preventing hepatitis A if given within 2 weeks after exposure. It may not prevent infection in all persons, but it will at least modify the illness to a subclinical infection. The hepatitis vaccine is only used for preexposure prophylaxis.

The nurse is obtaining a history for a 23-year-old woman who is being evaluated for acute lower abdominal pain and vomiting. Which question will be most useful in determining the cause of the patient's symptoms? A) "What is your usual elimination pattern?" B) "Can you tell me more about the pain?" C) "What type of foods do you usually eat?" D) "Is it possible that you are pregnant?"

B) "Can you tell me more about the pain?" Feedback: A complete description of the pain provides clues about the cause of the problem. The usual diet and elimination patterns are less helpful in determining the reason for the patient's symptoms. Although the nurse should ask whether the patient is pregnant to determine whether the patient might have an ectopic pregnancy and before any radiology studies are done, this information is not the most useful in determining the cause of the pain.

The nurse is preparing to insert a nasogastric tube into a 68-year-old patient with an abdominal mass and suspected bowel obstruction. The patient asks the nurse why this procedure is necessary. Which of the following responses is most appropriate? A) "The tube is just a standard procedure before many types of surgery to the abdomen." B) "The tube will help to drain the stomach contents and prevent further vomiting." C) "The tube will push past the area that is blocked, and thus help to stop the vomiting." D) "The tube will let us measure your stomach contents, so that we can plan what type of IV fluid replacement would be best."

B) "The tube will help to drain the stomach contents and prevent further vomiting." Feedback: The nasogastric tube is used to decompress the stomach by draining stomach contents, and thereby prevent further vomiting.

A patient with acute pancreatitis has a nasogastric (NG) tube to suction and is NPO. Which information obtained by the nurse indicates that these therapies have been effective? A) Bowel sounds are present. B) Abdominal pain is decreased. C) Grey Turner sign resolves. D) Electrolyte levels are normal.

B) Abdominal pain is decreased. Feedback: NG suction and NPO status will decrease the release of pancreatic enzymes into the pancreas and decrease pain. Although bowel sounds may be hypotonic with acute pancreatitis, the presence of bowel sounds does not indicate that treatment with NG suction and NPO status have been effective. Electrolyte levels will be abnormal with NG suction and must be replaced by appropriate IV infusion. Although Grey Turner sign will eventually resolve, it would not be appropriate to wait for this occur to determine whether treatment was effective.

A patient who has suffered severe burns in a motor vehicle accident will soon be started on parenteral nutrition (PN). Which of the following principles should guide the nurse's administration of the patient's nutrition? A) Administration of PN requires clean technique. B) Central PN requires rapid dilution in a large volume of blood. C) Only water-soluble medications may be added to the PN by the nurse. D) Peripheral PN delivery is preferred over the use of a central line.

B) Central PN requires rapid dilution in a large volume of blood. Feedback: Central PN is hypertonic and requires rapid dilution in a large volume of blood. Because PN is an excellent medium for microbial growth, aseptic technique is necessary during administration. Administration through a central line is preferred over the use of peripheral PN and no medications may be added to PN by the nurse.

The nurse is caring for a patient treated with intravenous fluid therapy for severe vomiting. As the patient recovers and begins to tolerate oral intake, the nurse understands that which of the following food choices would be most appropriate? A) Warm broth B) Dry toast C) Plain hamburger D) Ice tea

B) Dry toast Feedback: Dry toast or crackers may alleviate the feeling of nausea and prevent further vomiting. Extremely hot or cold liquids and fatty foods are generally not well tolerated.

The nurse will plan to teach the patient with newly diagnosed achalasia that A) A liquid or blenderized diet will be necessary. B) Endoscopic procedures may be used for treatment. C) Drinking fluids with meals should be avoided. D) Lying down and resting after meals is recommended.

B) Endoscopic procedures may be used for treatment. Feedback: Endoscopic and laparoscopic procedures are the most effective therapy for improving symptoms caused by achalasia. Patients are advised to drink fluid with meals. Keeping the head elevated after eating will improve esophageal emptying. A semisoft diet is recommended to improve esophageal emptying.

A patient who has dysphagia as a consequence of a stroke is receiving enteral feedings through a percutaneous endoscopic gastrostomy (PEG). Which of the following interventions should the nurse integrate into this patient's care? A) Flush the PEG with 100 ml of sterile water before and after medication administration. B) Flush the tube before and after feedings if the patient's feedings are intermittent. C) To prevent fluid overload, avoid flushing when the patient is receiving continuous feeding. D) Flush the tube with 30 ml of normal saline every 4 hours.

B) Flush the tube before and after feedings if the patient's feedings are intermittent. Feedback: The nurse should flush feeding tubes with 30 ml of water (not normal saline) every 4 hours during continuous feeding or before and after intermittent feeding. Flushes of 100 ml are excessive and may cause fluid overload in the patient.

When planning care for a patient with cirrhosis, the nurse will give highest priority to which of the following nursing diagnoses? A) Excess fluid volume related to portal hypertension and hyperaldosteronism B) Ineffective breathing pattern related to pressure on diaphragm and reduced lung volume C) Imbalanced nutrition: less than body requirements D) Impaired skin integrity related to edema, and ascites

B) Ineffective breathing pattern related to pressure on diaphragm and reduced lung volume Feedback: Although all of these nursing diagnoses are appropriate and important in the care of a patient with cirrhosis, airway and breathing are always the highest priorities.

A homeless patient with severe anorexia and fatigue is admitted to the hospital with viral hepatitis. Which patient goal has the highest priority when the nurse is developing the plan of care? A) Identify the source of exposure to hepatitis. B) Maintain adequate nutrition. C) Increase activity level. D) Establish a stable home environment.

B) Maintain adequate nutrition. Feedback: The highest priority outcome is to maintain nutrition because adequate nutrition is needed for hepatocyte regeneration. Finding a home for the patient and identifying the source of the infection would be appropriate activities, but they do not have as high a priority as ensuring adequate nutrition. Although the patient's activity level will be gradually increased, rest is indicated during the acute phase of hepatitis.

Which nursing action will the nurse include in the plan of care when admitting a patient with an exacerbation of inflammatory bowel disease (IBD)? A) Increase dietary fiber intake. B) Monitor stools for blood. C) Restrict oral fluid intake. D) Ambulate four times daily.

B) Monitor stools for blood. Feedback: Since anemia or hemorrhage may occur with IBD, stools should be assessed for the presence of blood. The other actions would not be appropriate for the patient with IBD. Because dietary fiber may increase gastrointestinal (GI) motility and exacerbate the diarrhea, severe fatigue is common with IBD exacerbations, and dehydration may occur.

When the nurse is assessing the mouth of a patient who uses smokeless tobacco for signs of oral cancer, which finding will be of most concern? A) Painful blisters at the border of the lips B) Red, velvety patches on the buccal mucosa C) Bleeding during tooth brushing D) White, curdlike plaques on the posterior tongue

B) Red, velvety patches on the buccal mucosa Feedback: A red, velvety patch suggests erythroplasia, which has a high incidence (greater than 50%) of progression to squamous cell carcinoma. The other lesions are suggestive of acute processes (gingivitis, oral candidiasis, and herpes simplex).

A patient who has just been started on continuous tube feedings of a full-strength commercial formula at 100 mL/hr using a closed system method has six diarrhea stools the first day. Which action should the nurse plan to take? A) Check gastric residual volumes more frequently. B) Slow the infusion rate of the tube feeding. C) Change the enteral feeding system and formula every 8 hours. D) Discontinue administration of water through the feeding tube.

B) Slow the infusion rate of the tube feeding. Feedback: Loose stools indicate poor absorption of nutrients and indicate a need to slow the feeding rate or decrease the concentration of the feeding. Water should be given when patients receive enteral feedings to prevent dehydration. When a closed enteral feeding system is used, the tubing and formula are changed every 24 hours. High residual volumes do not contribute to diarrhea.

The teaching plan for the patient being discharged following an acute episode of upper GI bleeding will include information concerning the importance of (select all that apply) a. only taking aspirin with milk or bread products b. avoiding taking aspirin and drugs containing aspirin c. taking only drugs prescribed by the health care provider d. taking all drugs 1 hour before mealtime to prevent further bleeding e. reading all OTC drug labels to avoid those containing stearic acid and calcium

B,C

The nurse would recognize that the liver performs which of the following functions (select all that apply)? A) a. Bile storage B) b. Detoxification C) c. Red blood cell (RBC) destruction D) d. Protein metabolism E) e. Steroid metabolism

B,C,D,E

A patient who has undergone an esophagectomy for esophageal cancer develops increasing pain, fever, and dyspnea when a full liquid diet is started postoperatively. The nurse recognizes that these symptoms are most indicative of: a. an intolerance to the feedings b. extension of the tumor into the aorta c. leakage of fluid or foods into the mediastinum d. esophageal perforation with fistula formation into the lung

C

Following administration of a dose of metoclopramide (Reglan) to the patient, the nurse determines that the medication has been effective when which of the following is noted? A) a. Decreased blood pressure B) b. Absence of muscle tremors C) c. Relief of nausea and vomiting D) d. No further episodes of diarrhea

C

In developing a weight reduction program with a 45-year-old patient who weighs 186 lb, the nurse encourages the patient to set a weight loss goal of how many pounds in 4 weeks? A) a. 1-2 B) b. 3-5 C) c. 4-8 D) d. 5-10

C

In developing an effective weight reduction plan for an overweight patient who states a willingness to try to lose weight, it is most important for the nurse to first assess which of the following factors? A) a. The length of time the patient has been obese B) b. The patient's current level of physical activity C) c. The patient's social, emotional, and behavioral influences on obesity D) d. Anthropometric measurements, such as body mass index and skinfold thickness

C

The health care team is assessing a patient for acute pancreatitis after he presented to the emergency department with severe abdominal pain. Which of the following laboratory values are the best diagnostic indicators of acute pancreatitis? A) a. Gastric pH B) b. Blood glucose C) c. Serum amylase D) d. Serum potassium

C

The nurse determines that a patient has experienced the beneficial effects of medication therapy with famotidine (Pepcid) when which of the following symptoms is relieved? A) a. Nausea B) b. Belching C) c. Epigastric pain D) d. Difficulty swallowing

C

The nurse is caring for a patient admitted to the hospital for asthma who weighs 186 lb (84.5 kg). During dietary counseling, the patient asks the nurse how much protein he should ingest each day. How many grams of protein does the nurse recommend should be included in the diet based on the patient's current weight? A) a. 24 B) b. 41 C) c. 68 D) d. 93

C

The nurse is involved in health promotion related to oral cancer. Teaching young adults about behaviors that put them at risk for oral cancer includes: a. discouraging use of chewing gum b. avoiding use of perfumed lip gloss c. avoiding use of smokeless tobacco d. discouraging drinking of carbonated beverages

C

The nurse is providing care for a 23-year-old woman who is a strict vegetarian. To prevent the consequences of iron deficiency, the nurse should recommend A) a. Brown rice and kidney beans. B) b. Cauliflower and egg substitutes. C) c. Soybeans and hot breakfast cereal. D) d. Whole-grain bread and citrus fruits.

C

The nurse recognizes that the majority of patients' caloric needs should come from which of the following sources? A) a. Fats B) b. Proteins C) c. Polysaccharides D) d. Monosaccharides

C

The patient receiving chemotherapy rings the call bell and reports an onset of nausea. The nurse should prepare a prn dose of which of the following medications? A) a. Morphine sulfate B) b. Zolpidem (Ambien) C) c. Ondansetron (Zofran) D) d. Dexamethasone (Decadron)

C

The pernicious anemia that may accompany gastritis is due to which of the following? a. chronic autoimmune destruction of cobalamin stores in the body b. progressive gastric atrophy from chronic breakage in the mucosal barrier and blood loss c. a lack of intrinsic factor normally produced by acid-secreting cells in the gastric mucosa d. hyperchlorhydria resulting from an increase in acid-secreting parietal cells and degradations of RBCs

C

When assessing the health perception-health maintenance pattern as related to GI function, an appropriate question by the nurse is: a. what is your usual bowel elimination pattern? b. what percentage of your income is spent on food? c. have you traveled to a foreign country in the last year? d. do you have diarrhea when you are under a lot of stress?

C

Which of the following statements best described the etiology of obesity: a. obesity primarily results from a genetic predisposition b. psychosocial factors can override the effects of genetics in the etiology of obesity c. Obesity is the result of complex interactions between genetic and environmental factors d. genetic factors are more important than environmental factors in the etiology of obesity

C

Which of these laboratory test results will be most important for the nurse to monitor when evaluating the effects of therapy for a patient who has acutepancreatitis? A) Bilirubin B) Potassium C) Amylase D) Calcium

C) Amylase Feedback: Amylase is elevated in acute pancreatitis. Although changes in the other values may occur, they would not be as useful in evaluating whether the prescribed therapies have been effective.

A 67-year-old patient tells the nurse, ""I have problems with constipation now that I am older, so I use a suppository every morning."" Which action should the nurse take first? A) Suggest that the patient increase dietary intake of high-fiber foods. B) Inform the patient that a daily bowel movement is unnecessary. C) Assess the patient about individual risk factors for constipation. D) Encourage the patient to increase oral fluid intake.

C) Assess the patient about individual risk factors for constipation. Feedback: The nurse's initial action should be further assessment of the patient for risk factors for constipation and for usual bowel pattern. The other actions may be appropriate but will be based on the assessment.

After the nurse teaches a patient with gastroesophageal reflux disease (GERD) about recommended dietary modifications, which diet choice for a snack 2 hours before bedtime indicates that the teaching has been effective? A) Glass of low-fat milk B) Peanut butter sandwich C) Cherry gelatin and fruit D) Chocolate pudding

C) Cherry gelatin and fruit Feedback: Gelatin and fruit are low fat and will not decrease lower esophageal sphincter (LES) pressure. Foods like chocolate are avoided because they lower LES pressure. Milk products increase gastric acid secretion. High-fat foods such as peanut butter decrease both gastric emptying and LES pressure.

After a patient has had a transjugular intrahepatic portosystemic shunt (TIPS) placement, which finding indicates that the procedure has been effective? A) Increase in serum albumin level B) Lower indirect bilirubin level C) Decrease in episodes of variceal bleeding D) Improvement in alertness and orientation

C) Decrease in episodes of variceal bleeding Feedback: TIPS is used to lower pressure in the portal venous system and decrease the risk of bleeding from esophageal varices. Indirect bilirubin level and serum albumin levels are not affected by shunting procedures. TIPS will increase the risk for hepatic encephalopathy.

A patient is hospitalized with vomiting of ""coffee-ground"" emesis. The nurse will anticipate preparing the patient for A) Barium contrast studies. B) Gastric analysis testing. C) Endoscopy. D) Angiography.

C) Endoscopy. Feedback: Endoscopy is the primary tool for visualization and diagnosis of upper gastrointestinal (GI) bleeding. Angiography is used only when endoscopy cannot be done because it is more invasive and has more possible complications. Gastric analysis testing may help with determining the cause of gastric irritation, but it is not used for acute GI bleeding. Barium studies are helpful in determining the presence of gastric lesions, but not whether the lesions are actively bleeding.

A critically ill patient develops incontinence of watery stools. What action will be best for the nurse to take to prevent complications associated with ongoing incontinence? A) Assist the patient to a bedside commode or to the bathroom at frequent intervals. B) Use incontinence briefs. C) Implement fecal management system. D) Insert a rectal tube.

C) Implement fecal management system. Feedback: Fecal management systems are designed to contain loose stools and can be in place for as long as 4 weeks without causing damage to the rectum or anal sphincters. Rectal tubes are avoided because of possible damage to the anal sphincter and ulceration of the rectal mucosa. Incontinence briefs may be helpful but, unless they are changed frequently, are likely to increase the risk for skin breakdown. A critically ill patient will not be able to use the commode or bathroom.

The nurse would question the use of which of the following cathartic agents in a patient with renal insufficiency? A) Bisacodyl B) Cascara sagrada C) Milk of magnesia D) Lubiprostone

C) Milk of magnesia Feedback: Milk of magnesia may cause hypermagnesemia in patients with renal insufficiency. The nurse should question this order with the health care provider before administration.

A patient with a gunshot wound to the abdomen undergoes surgery, and a colostomy is formed as illustrated. Which information will be included in patient teaching? A) Irrigations can regulate drainage from the stomas. B) Soft, formed stool can be expected as drainage. C) This type of colostomy is usually temporary. D) Stool will be expelled from both ostomystomas.

C) This type of colostomy is usually temporary. Feedback: A loop, or double-barrel stoma, is usually temporary. Stool will be expelled from the proximal stoma only. The stool from the transverse colon will be liquid and regulation through irrigations will not be possible.

A morbidly obese patient has undergone Roux-en-Y gastric bypass surgery. In planning postoperative care, the nurse anticipates that the patient a. may have severe diarrhea early in the postoperative period b. will not be allowed to ambulate for 1 or 2 days postoperatively c. will require nasogastric suction until healing of the incision occurs d. may have only liquids orally, and in very limited amounts, during the early postoperative period

D

A patient is admitted to the hospital with a diagnosis of diarrhea with dehydration. The nurse recognized that increased peristalsis resulting in diarrhea can be related to: a. sympathetic inhibition b. mixing and propulsion c. sympathetic stimulation d. parasympathetic stimulation c

D

A patient reports having dry mouth and asks for some liquid to drink. The nurse reasons that this symptom can most likely be attributed to a common adverse effect of which of the following medications? A) a. Digoxin (Lanoxin) B) b. Cefotetan (Cefotan) C) c. Famotidine (Pepcid) D) d. Promethazine (Phenergan)

D

A patient with anorexia nervosa shows signs of malnutrition. During initial refeeding, the nurse carfully assesses the patient for: a. hyperkalemia b. hypoglycemia c. hypercalcemia d. hypophosphatemia

D

An optimal teaching plan for an outpatient with stomach cancer receiving radiation therapy should include information about a. cancer support groups, alopecia, and stomatitis b, avitaminosis, ostomy care, and community resources c. prosthetic devices, skin conductance, and grief counseling d. wound and skin care, nutrition, drugs, and community resources

D

The best nutritional therapy plan for a person who is obese is: a. the Zone diet b. the Atkins diet c. Sugar busters d. foods from the basic food groups

D

The nurse explains to the patient with Vincent's infection that treatment will include: a. smallpox vaccinations b. viscous lidocaine rinses c. amphotericin B suspension d. topical application of antibiotics

D

The nurse explains to the patient with gastroesophageal reflux disease that this disorder: a. results in acid erosion and ulceration of the esophagus caused by frequent vomiting b. will require surgical wrapping or repair of the pyloric sphincter to control the symptoms c. is the protrusion of a portion of the stomach into the esophagus through an opening in the diaphragm d. often involves relation of the lower esophageal sphincter, allowing stomach contents to back up into the esophagus

D

The nurse is performing a focused abdominal assessment of a patient who has been recently admitted. In order to palpate the patient's liver, the nurse will palpate the patient's A) a. Left lower quadrant. B) b. Left upper quadrant. C) c. Right lower quadrant. D) d. Right upper quadrant.

D

The nurse is reviewing the laboratory test results for a 71-year-old patient with metastatic lung cancer. The patient was admitted with a diagnosis of malnutrition. Serum albumin level is 4.0 g/dl and prealbumin is 10 mg/dl. The nurse should draw which of the following conclusions? A) a. The albumin level is normal, and therefore the patient does not have protein malnutrition. B) b. The albumin level is increased, which is a common finding in patients with cancer who have malnutrition. C) c. Both the serum albumin and prealbumin levels are reduced, consistent with the admitting diagnosis of malnutrition. D) d. Although the serum albumin level is normal, the prealbumin level more accurately reflects the patient's nutritional status.

D

The nurse is teaching the patient and family about possible causative factors for peptic ulcers. The nurse explains that ulcer formation is: a. caused by a stressful lifestyle and other acid producing factors such as h.pylori b. inherited within families and reinforced by bacterial spread of staph aureus in childhood c. promoted by factors that tend to cause oversecretion of acid such as excess dietary fats, smoking and h.pylori d. promoted by a combination of possible factors that may result in erosion of the gastric mucosa, including certain drugs and alcohol

D

The obesity classification that is most often associated with cardiovascular health problems is: a. primary obesity b. secondary obesity c. gynoid fat distribution d. android fat distribution

D

The optimal way to administer medications via a feeding tube is to: a. use only sustained-release medications b. pour medications into the enteral formula c. grind all the pills together and dilute with water d. remover powder from a gelatin capsule and dilute with water

D

This bariatric surgical procedure involves creating a stoma and gastric pouch that is reversible and no malabsorption occurs. What surgical procedure is this? a. vertical gastric banding b. biliopacreatic diversion c. roux-en-y gastric bypass d. adjustable gastric banding

D

A patient is admitted with an abrupt onset of jaundice, nausea, and abnormal liver function studies. Serologic testing is negative for viral causes of hepatitis. Which question by the nurse is most appropriate? A) "Are you taking corticosteroids for any reason?" B) "Is there any history of IV drug use?" C) "Have you recently traveled to a foreign country?" D) "Do you use any over-the-counter (OTC) drugs?"

D) "Do you use any over-the-counter (OTC) drugs?" Feedback: The patient's symptoms, lack of antibodies for hepatitis, and the abrupt onset of symptoms suggest toxic hepatitis, which can be caused by commonly used OTC drugs such as acetaminophen (Tylenol). Travel to a foreign country and a history of IV drug use are risk factors for viral hepatitis. Corticosteroid use does not cause the symptoms listed.

The nurse explains to a patient with a new ileostomy that after the bowel adjusts to the ileostomy, the usual drainage will be about A) 1 cup. B) 1 quart. C) 3 cups. D) 2 cups.

D) 2 cups. Feedback: After the proximal small bowel adapts to reabsorb more fluid, the average amount of ileostomy drainage is about 500 mL daily.

After receiving change-of-shift report, which patient should the nurse assess first? A) A patient who is crying after receiving a diagnosis of esophageal cancer B) A patient who was admitted yesterday with gastrointestinal (GI) bleeding and has melena C) A patient with nausea who has a dose of metoclopramide (Reglan) scheduled D) A patient with esophageal varices who has a blood pressure of 96/54 mm Hg

D) A patient with esophageal varices who has a blood pressure of 96/54 mm Hg Feedback: The patient's history and blood pressure indicate possible hemodynamic instability caused by GI bleeding. The data about the other patients do not indicate acutely life-threatening complications.

A patient is receiving tube feedings through a percutaneous endoscopic gastrostomy (PEG). Which action will the nurse include in the plan of care? A) Avoid giving bolus tube feedings through the PEG tube. B) Keep the patient positioned on the left side. C) Obtain a daily x-ray to verify tube placement. D) Check the gastric residual volume every 4 to 6 hours. A patient is admitted with anorexia nervosa and a serum potassium level of 2.4 mEq/L. What is most important for the nurse to observe for in this patient? A) Cardiac dysrhythmias B) Muscle weakness C) Increased urine output D) Anemia and leukopenia

D) Check the gastric residual volume every 4 to 6 hours. Feedback: The gastric residual volume is assessed every 4 to 6 hours to decrease the risk for aspiration. The patient does not need to be positioned on the left side. An x-ray is obtained immediately after placement of the PEG tube to check position, but daily x-rays are not needed. Bolus feedings can be administered through a PEG tube.

Which of these nursing activities included in the care of a patient with a new colostomy should the nurse delegate to nursing assistive personnel (NAP)? A) Document the appearance of the stoma. B) Place the pouching system over the ostomy. C) Check the skin around the ostomy for breakdown. D) Drain and measure the output from the ostomy.

D) Drain and measure the output from the ostomy. Feedback: Draining and measuring the output from the ostomy is included in NAP education and scope of practice. The other actions should be implemented by LPNs or RNs.

All of the following orders are received for a patient who has vomited 1500 mL of bright red blood. Which order will the nurse implement first? A) Insert a nasogastric (NG) tube and connect to suction. B) Draw blood for typing and crossmatching. C) Administer intravenous (IV) famotidine (Pepcid) 40 mg. D) Infuse 1000 mL of lactated Ringer's solution.

D) Infuse 1000 mL of lactated Ringer's solution. Feedback: Because the patient has vomited a large amount of blood, correction of hypovolemia and prevention of hypovolemic shock are the priorities. The other actions also are important to implement quickly but are not the highest priorities.

Which of these prescribed interventions will the nurse implement first when caring for a patient who has just been diagnosed with peritonitis caused by a ruptured diverticulum? A) Insert a nasogastric (NG) tube and connect it to intermittent low suction. B) Administer morphine sulfate 4 mg IV. C) Send the patient for a computerized tomography scan. D) Infuse metronidazole (Flagyl) 500 mg IV.

D) Infuse metronidazole (Flagyl) 500 mg IV. Feedback: Since peritonitis can be fatal if treatment is delayed, the initial action should be to start antibiotic therapy (after any ordered cultures are obtained). The other actions can be done after antibiotic therapy is initiated.

A patient with malnutrition is on a high-protein, high-calorie diet. Which item would be the most appropriate for the nurse to provide for this patient? A) Orange juice and dry toast B) Steamed carrots and chicken broth C) Banana and unsweetened applesauce D) Oatmeal, butter, and cream

D) Oatmeal, butter, and cream Feedback: Rationale: Oatmeal, butter, and cream are examples of food items that would be appropriate to include for a patient on a high-protein, high-calorie diet. See Table 40-11 for other examples.

A patient with a peptic ulcer who has a nasogastric (NG) tube develops sudden, severe upper abdominal pain, diaphoresis, and a very firm abdomen. Which action should the nurse take next? A) Give the ordered antacid. B) Listen for bowel sounds. C) Irrigate the NG tube. D) Obtain the vital signs.

D) Obtain the vital signs. Feedback: The patient's symptoms suggest acute perforation, and the nurse should assess for signs of hypovolemic shock. Irrigation of the NG tube, administration of antacids, or both would be contraindicated because any material in the stomach will increase the spillage into the peritoneal cavity. The nurse should assess the bowel sounds, but this is not the first action that should be taken.

All of the following nursing actions are included in the plan of care for a patient who is malnourished. Which action is appropriate for the nurse to delegate to nursing assistive personnel (NAP)? A) Monitor the patient for skin breakdown over the bony prominences. B) Assess the patient's strength while ambulating the patient in the room. C) Assist the patient to choose high nutrition items from the menu. D) Offer the patient the prescribed nutritional supplement between meals.

D) Offer the patient the prescribed nutritional supplement between meals. Feedback: Feeding the patient and assisting with oral intake are included in NAP education and scope of practice. Assessing the patient and assisting the patient in choosing high nutrition foods require LPN/LVN- or RN-level education and scope of practice.

A patient reports having dry mouth and asks for some liquid to drink. The nurse reasons that this symptom can most likely be attributed to a common adverse effect of which of the following medications? A) Famotidine (Pepcid) B) Digoxin (Lanoxin) C) Cefotetan (Cefotan) D) Promethazine (Phenergan)

D) Promethazine (Phenergan) Feedback: A common adverse effect of promethazine, an antihistamine antiemetic agent, is dry mouth; another is blurred vision.

Which of the following nursing actions included in the plan of care for a patient with cirrhosis can the RN delegate to nursing assistive personnel? A) Palpating the abdomen for distention B) Assisting the patient in choosing the diet C) Assessing the patient for jaundice D) Providing oral hygiene before meals

D) Providing oral hygiene before meals Feedback: Providing oral hygiene is included in the education and scope of practice of nursing assistants. Assessments and assisting patients to choose therapeutic diets are nursing actions that require higher-level nursing education and scope of practice and would be delegated to LPNs/LVNs or RNs.

After 6 hours of parenteral nutrition (PN) infusion, the nurse checks a patient's capillary blood glucose level and finds it to be 120 mg/dL. The most appropriate action by the nurse is to A) Notify the health care provider of the glucose level. B) Obtain a venous blood glucose specimen. C) Slow the infusion rate of the PN infusion. D) Recheck the capillary blood glucose in 4 hours.

D) Recheck the capillary blood glucose in 4 hours. Feedback: Mild hyperglycemia is expected during the first few days after PN is started and requires ongoing monitoring. Because the glucose elevation is small and expected, notification of the health care provider is not necessary. There is no need to obtain a venous specimen for comparison. Slowing the rate of the infusion is beyond the nurse's scope of practice and will decrease the patient's nutritional intake.

The nurse is providing care for a 23-year-old woman who is a strict vegetarian. To prevent the consequences of iron deficiency, the nurse should recommend A) Whole-grain bread and citrus fruits. B) Cauliflower and egg substitutes. C) Brown rice and kidney beans. D) Soybeans and hot breakfast cereal.

D) Soybeans and hot breakfast cereal. Feedback: Vegetarians are at a particular risk for iron deficiency, a problem that can be prevented by regularly consuming high-iron foods such as hot cereals and soybeans. The other foods listed are not classified as high sources of iron.

A patient with acute gastrointestinal (GI) bleeding is receiving normal saline IV at a rate of 500 mL/hr. Which assessment finding obtained by the nurse is most important to communicate immediately to the health care provider? A) The patient's blood pressure (BP) has increased to 142/94 mm Hg. B) The nasogastric (NG) suction is returning coffee-ground material. C) The bowel sounds are very hyperactive in all four quadrants. D) The patient's lungs have crackles audible to the midline.

D) The patient's lungs have crackles audible to the midline. Feedback: The patient's lung sounds indicate that pulmonary edema may be developing as a result of the rapid infusion of IV fluid and that the fluid infusion rate should be slowed. The return of coffee-ground material in an NG tube is expected for a patient with upper GI bleeding. The BP is slightly elevated but would not be an indication to contact the health care provider immediately. Hyperactive bowel sounds are common when a patient has GI bleeding.

1.The nurse is reviewing the laboratory test results for a 71-year-old patient with metastatic lung cancer. The patient was admitted with a diagnosis of malnutrition. Serum albumin level is 4.0 g/dl and prealbumin is 10 mg/dl. The nurse should draw which of the following conclusions? A) The albumin level is normal, and therefore the patient does not have protein malnutrition. B) The albumin level is increased, which is a common finding in patients with cancer who have malnutrition. C) Both the serum albumin and prealbumin levels are reduced, consistent with the admitting diagnosis of malnutrition. D) Although the serum albumin level is normal, the prealbumin level more accurately reflects the patient's nutritional status.

Serum albumin has a half-life of approximately 20 to 22 days; therefore the serum level may lag behind actual protein changes by more than 2 weeks and is therefore not a good indicator of acute changes in nutritional status. Prealbumin has a half-life of 2 days and is a better indicator of recent or current nutritional status. Points Earned: 0.0/1.0 Correct Answer(s): D


Conjuntos de estudio relacionados

Econ 201- Chapter 1 Quiz and Homework

View Set

Ch. 1 Sec. 2 Examining Your Personality

View Set

Chem exam #3 connect homework part two

View Set

Psychology 6.3 Operant Conditioning terms

View Set

Cost: Chapter 11 HW - Decision Making

View Set